What is the linear approximation for estimating f(x,y)?

Click For Summary
The discussion focuses on using linear approximation to estimate the function f(x,y) = √((4.01)² + (3.98)² + (2.02)²). The initial approach involves setting x = 4 and y = 2, but it is clarified that the function actually has three variables, requiring the use of partial derivatives. The correct formula for linear approximation is provided, emphasizing the need to evaluate the function at specific points and calculate the changes in each variable. Despite initial confusion regarding the computed differential being zero, it is confirmed that the approximation is valid and close to the exact value. The final estimated value aligns well with the exact calculation, demonstrating the effectiveness of the linear approximation method.
Iconate
Messages
20
Reaction score
0

Homework Statement


Use the linear approximation to approximate a suitable function f(x,y) and thereby estimate the following

f(x,y) = \sqrt{(4.01)^2 + (3.98)^2 + (2.02)^2}

Homework Equations



Not going to type it out, but the formula for f(x,y) http://en.wikipedia.org/wiki/Linear_approximation

The Attempt at a Solution



I just need to find the equation first, I can do the estimation.

My guess is, let x = 4, and y = 2 then

f(x,y) = \sqrt{(x)^2 + (x)^2 + (y)^2}
=(2(x)2 + (y)2)1/2

Is that done correctly? Proceeding this would be to solve the Linear Approximation formula, and check to see that it is infact close to \sqrt{(4.01)^2 + (3.98)^2 + (2.02)^2}

Any help would be great thanks
 
Physics news on Phys.org
Iconate said:

Homework Statement


Use the linear approximation to approximate a suitable function f(x,y) and thereby estimate the following

f(x,y) = \sqrt{(4.01)^2 + (3.98)^2 + (2.02)^2}

Homework Equations



Not going to type it out, but the formula for f(x,y) http://en.wikipedia.org/wiki/Linear_approximation

The Attempt at a Solution



I just need to find the equation first, I can do the estimation.

My guess is, let x = 4, and y = 2 then

f(x,y) = \sqrt{(x)^2 + (x)^2 + (y)^2}
=(2(x)2 + (y)2)1/2

Is that done correctly? Proceeding this would be to solve the Linear Approximation formula, and check to see that it is infact close to \sqrt{(4.01)^2 + (3.98)^2 + (2.02)^2}

Any help would be great thanks
Your function is actually one with three variables, f(x, y, z), and its formula is sqrt(x^2 + y^2 + z^2)

Here's what you want to use:
f(x0 + dx, y0 + dy, z0 + dy) \approx f(x0, y0, z0) + df
where df = fx(x0, y0, z0)*dx + fy(x0, y0, z0)*dy + fz(x0, y0, z0)*dz.

The notation fx(x0, y0, z0) means the partial deriviative of f, evaluated at (x0, y0, z0), and so on for the other two partials.

For your problem, x0 = 4, y0 = 4, and z0 = 2
dx = .01, dy = -.02, and dz = .02
 
Ah, That all makes sense, but when I computed df, I got that equal to 0

df/dx = x/\sqrt{(x)^2 + (z)^2 + (y)^2} = 0.6666...
df/dy = 0.6666...
df/dz = 0.3333...

which means, my approximation is exact, which i don't think is right >.<
 
Last edited:
It just means that df = 0.
f(x0 + dx, y0 + dy, z0 + dy) \approx
f(x0, y0, z0) + df
The fact that df = 0 doesn't turn the above into an equality.

In this case, the exact value is sqrt(36.009) \approx 6.000075, which is approximately equal to your estimate, sqrt(36) = 6
 
Ohh I see perfect, thank you very much
 
Question: A clock's minute hand has length 4 and its hour hand has length 3. What is the distance between the tips at the moment when it is increasing most rapidly?(Putnam Exam Question) Answer: Making assumption that both the hands moves at constant angular velocities, the answer is ## \sqrt{7} .## But don't you think this assumption is somewhat doubtful and wrong?

Similar threads

Replies
2
Views
2K
  • · Replies 3 ·
Replies
3
Views
2K
  • · Replies 3 ·
Replies
3
Views
2K
Replies
6
Views
2K
  • · Replies 3 ·
Replies
3
Views
2K
  • · Replies 1 ·
Replies
1
Views
1K
  • · Replies 7 ·
Replies
7
Views
2K
  • · Replies 3 ·
Replies
3
Views
2K
  • · Replies 11 ·
Replies
11
Views
2K
  • · Replies 12 ·
Replies
12
Views
3K